• Asignatura: Matemáticas
  • Autor: juaneslozano1006
  • hace 4 años

Escribir los términos que hacen verdadera la igualdad:
__ x (-3) x (__) = 9

(-9) x __ x (-1) = 36


ayuda plis :,,c

Respuestas

Respuesta dada por: kt192
0

Respuesta:2(x + 1) - 3(x - 2) < x + 6

Solución

Procedamos a resolver la inecuación, recuerda que es muy parecido a un despeje en una igualdad, solo que ahora en vez de encontrar únicamente un valor para nuestra variable, encontramos todo un dominio, muchas veces formado por un intervalo o por uniones o intercepciones de intervalos. Nuestra inecuación es

 

2(x + 1) - 3(x - 2) < x + 6

 

Procedamos, para esto primero haremos uso de la propiedades distributiva y luego despejaremos x

 

 \begin{align*} 2(x + 1) - 3(x - 2) &< x + 6\\2x + 2 - 3x + 6 &< x + 6\\-x + 8 &< x + 6\\2 &< 2x\\1 &< x\end{align*}

 

Notemos que esto nos dice que 1 < x, o bien, tenemos que su conjunto de solución es el intervalo (1, \infty).

 

Gráfica de valores que cumplen la inecuacion

 

2\displaystyle \frac{3x + 1}{7} - \frac{2 - 4x}{3} \geq \frac{-5x - 4}{14} + \frac{7x}{6}

Solución

Nuestra inecuación a resolver es la siguiente:

 

\displaystyle \frac{3x + 1}{7} - \frac{2 - 4x}{3} \geq \frac{-5x - 4}{14} + \frac{7x}{6}

 

Procedamos, para esto nos desharemos de las fracciones primero y luego despejaremos x. Para deshacernos de las fracciones necesitamos el mínimo común múltiplo de los denominadores, el cual es \text{MCM}(7, 3, 14, 6) = 42, así, obtenemos

 

 \begin{align*} \frac{3x + 1}{7} - \frac{2 - 4x}{3} &\geq \frac{-5x - 4}{14} + \frac{7x}{6}\\\frac{42(3x + 1)}{7} - \frac{42(2 - 4x)}{3} &\geq \frac{42(-5x - 4)}{14} + \frac{42(7x)}{6}\\6(3x + 1) - 14(2 - 4x) &\geq 3(-5x - 4) + 7(7x)\\18x + 6 - 28 + 56x &\geq -15x - 12 + 49x\\74x -22 &\geq 34x - 12\\40x &\geq 10\\x &\geq \frac{1}{4}\end{align*}

 

Notemos que esto nos dice que \displaystyle x \geq \frac{1}{4}, o bien, tenemos que su conjunto de solución es el intervalo \displaystyle \left[\frac{1}{4}, \infty\right).

 

Gráfica de valores que cumplen la inecuacion

3\displaystyle 6 \left(\frac{x + 1}{8} - \frac{2x - 3}{16} \right) > 3\left( \frac{3}{4}x - \frac{1}{4}\right) - \frac{3}{8}(3x - 2)

Solución

Nuestra inecuación a resolver es la siguiente:

 

\displaystyle 6 \left(\frac{x + 1}{8} - \frac{2x - 3}{16} \right) > 3\left( \frac{3}{4}x - \frac{1}{4}\right) - \frac{3}{8}(3x - 2)

 

Procedamos, para esto, al igual que en los ejercicio anteriores, necesitamos aplicar ley distributiva y, además, eliminar las fracciones

 

  \begin{align*} 6 \left(\frac{x + 1}{8} - \frac{2x - 3}{16} \right) &> 3\left( \frac{3}{4}x - \frac{1}{4}\right) - \frac{3}{8}(3x - 2)\\\frac{3(x + 1)}{4} - \frac{3(2x - 3)}{8} &> \frac{9}{4}x - \frac{3}{4} - \frac{3(3x - 2)}{8}\\\frac{3x + 3}{4} - \frac{6x - 9}{8} &> \frac{9x}{4} - \frac{3}{4} - \frac{9x - 6}{8}\\\frac{8(3x + 3)}{4} - \frac{8(6x - 9)}{8} &> \frac{8(9)x}{4} - \frac{8(3)}{4} - \frac{8(9x - 6)}{8}\\2(3x + 3) - (6x - 9) &> 2(9)x - 2(3) - (9x - 6)\\6x + 6 - 6x + 9 &> 18x - 6 - 9x + 6\\15 &> 9x\\\frac{15}{9} &> x \\\frac{5}{3} &> x \\\end{align*}

 

Notemos que esto nos dice que x < \frac{5}{3}, o bien, tenemos que su conjunto de solución es el intervalo \displaystyle \left( -\infty, \frac{5}{3}\right).

 

4 \displaystyle \begin{cases}10(x + 1) + x \leq 6(2x + 1)\\4(x - 10) < -6(2 - x) - 6x\end{cases}

Solución

Para resolver este sistema debemos resolver las inecuaciones por serparado y luego encontrar los valores para los cuales x cumple ambas inecuaciones.

 

Empecemos con la primer inecuación

 

 \begin{align*} 10(x + 1) + x &\leq 6(2x + 1)\\10x + 10 + x &\leq 12x + 6\\11x + 10 &\leq 12x + 6\\4 &\leq x\end{align*}

 

así, de la primer inecuación tenemos que 4 \leq x, o bien, que x pertenece al intervalo [4, \infty). Ahora resolvamos la segunda inecuación

 

 \begin{align*} 4(x - 10) &< -6(2 - x) - 6x\\4x - 40 &< -12 + 6x - 6x\\4x - 40 &< -12 \\4x &< 28 \\x &< 7 \\\end{align*}

 

así, de la segunda inecuación tenemos que x < 7, o bien, que x pertenece al intervalo (-\infty, 7). Notemos que equis debe cumplir ambas condiciones, esto es, 4 \leq x y x < 7, esto es equivalente a decir que x se encuentra en la intersección de los intervalos [4, \infty) y (-\infty, 7), así, el conjunto solución es

 

\displaystyle [4, \infty) \cap (-\infty, 7) = [4, 7)

 

Gráfica de valores que cumplen la inecuacion

Explicación paso a paso:

Preguntas similares